× Didn't find what you were looking for? Ask a question
Top Posters
Since Sunday
14
o
6
6
G
3
c
3
q
3
m
3
j
3
s
2
b
2
j
2
u
2
New Topic  
tomato.3141 tomato.3141
wrote...
Posts: 46
Rep: 1 0
11 years ago
A proton moves with a velocity of = (1i - 1j + k) m/s in a region in which the magnetic field is = (i + 2j - 3k) T. What is the magnitude of the magnetic force this charge experiences?

I totally forgot cross product rule. Can you just solve the problem for me pleeease Slight Smile
 
I tried:
+q(-5i+4j+k)  but it did not work. Frowning Face
Read 517 times
1 Reply

Related Topics

Replies
wrote...
11 years ago
F = +q(-i - 4j - 3k)
New Topic      
Explore
Post your homework questions and get free online help from our incredible volunteers
  1119 People Browsing
Related Images
  
 216
  
 16
  
 571
Your Opinion
Which country would you like to visit for its food?
Votes: 261

Previous poll results: How often do you eat-out per week?